Solving Improper Integrals: ∫1/(x^2-1)dx from 2 to ∞

  • Thread starter morris4019
  • Start date
  • Tags
    Integrals
In summary, the conversation discusses a problem in Calculus 2 involving improper integrals. The person asks for clarification and confirmation on their solution, and another person helps them by providing hints and corrections. Eventually, the correct solution is reached, which involves taking the limit of a ratio of polynomials with the same degree.
  • #1
morris4019
3
0
Hi, I am currently in Calculus 2 at my local college and I am having trouble wrapping my head around Improper Integrals. The question below I have been working on for awhile and I think i have an answer but was wondering if anyone could confirm if I was thinking about this question the right way:


Homework Statement


Calculate ∫1/(x^2-1)dx from 2 to positive infinity. (Hint: You will need to write the antiderivative as a single logarithm in order to be able to calculate the appropriate limit.)


Homework Equations





The Attempt at a Solution





What I have so far is the following:

First re-writing as a limit->

lim ∫1/(x^2-1)dx from 2 to T
T->infinity

then using partial fractions

lim 1/2∫1/(x-1)dx - 1/2∫1/(x+1)dx from 2 to T
T->infinity

lim 1/2*ln|x-1| - 1/2*ln|x+1| from 2 to T
T->infinity

re-writing as a single natual log

lim 1/2*ln((x-1)/(x+1)) from 2 to T
T->infinity

now subtracting the endpoints

lim 1/2*ln((T-1)/(T+1)) - 1/2*ln(1/3)
T->infinity

now here is where i got a little confused again. T is approaching infinity but because we are taking a limit i can say that the first term is siply 1/2 correct? and the second remains 1/2*ln(1/3)? So I'm getting for my answers (exact and then approx):

1/2 - 1/2*ln(1/3) or approx 1.0493


Now I could be completely wrong and that is why I can't seem to get comfortable with these things. Can anyone shed some light on what I'm doing wrong, or perhaps right?

Thanks
-Mike
 
Physics news on Phys.org
  • #2
what do you know about taking the limit as x->inf of the ratio of polynomials with the same degree? Hint - has to do with the coefficients

Then what is the Ln of that?
 
  • #3
The limit of a ratio of polylnomials, with the same degree both top and bottom, should just equal the coefficient ratio, correct? so...

lim 2x^2 / 3x^2 = 2/3
x->inf

So my limit near the bottom:

lim 1/2*ln((T-1)/(T+1)) - 1/2*ln(1/3)
T->infinity

should be:

=> 1/2*(ln 1) - 1/2*ln(1/3)
=> 1/2*(0) - 1/2*ln(1/3)
=> -1/2*ln(1/3)

does this look a little better?
 
  • #4
Yes, That is correct.
 
  • #5
thank you, that makes more sense. I've spoken to a few people about what i did above and no one pointed out that I made that mistake with the natural log.
 

Related to Solving Improper Integrals: ∫1/(x^2-1)dx from 2 to ∞

What is an improper integral?

An improper integral is a type of integral where one or both of the limits of integration are infinite or where the integrand has an infinite discontinuity within the interval of integration. This makes the integral undefined and requires special techniques to evaluate.

Why is the integral ∫1/(x^2-1)dx from 2 to ∞ considered improper?

This integral is considered improper because the upper limit of integration is infinite. When x approaches infinity, the integrand becomes undefined, making it impossible to evaluate the integral using traditional methods.

How do you solve an improper integral?

To solve an improper integral, you need to use a technique called "limit of integration". This involves taking the limit of the integral as the upper and/or lower limits approach infinity or a point of discontinuity. In this case, we can use the limit as x approaches infinity to evaluate the integral.

What is the limit as x approaches infinity for ∫1/(x^2-1)dx?

As x approaches infinity, the integrand approaches 0. This means that the limit of the integral is equal to the limit of the original integrand, which is 0. So, the value of the integral is simply 0.

What is the final solution for the integral ∫1/(x^2-1)dx from 2 to ∞?

The final solution for this integral is 0. This can be found by taking the limit of the integral as x approaches infinity, which we know is equal to the limit of the integrand, which is 0. Therefore, the value of the integral is 0.

Similar threads

  • Calculus and Beyond Homework Help
Replies
14
Views
489
  • Calculus and Beyond Homework Help
2
Replies
54
Views
8K
  • Calculus and Beyond Homework Help
Replies
2
Views
378
  • Calculus and Beyond Homework Help
Replies
2
Views
500
  • Calculus and Beyond Homework Help
Replies
3
Views
1K
  • Calculus and Beyond Homework Help
Replies
3
Views
424
  • Calculus and Beyond Homework Help
Replies
12
Views
1K
  • Calculus and Beyond Homework Help
Replies
2
Views
358
  • Calculus and Beyond Homework Help
Replies
12
Views
1K
  • Calculus and Beyond Homework Help
Replies
5
Views
1K
Back
Top